LSAT and Law School Admissions Forum

Get expert LSAT preparation and law school admissions advice from PowerScore Test Preparation.

 Administrator
PowerScore Staff
  • PowerScore Staff
  • Posts: 8916
  • Joined: Feb 02, 2011
|
#73472
Complete Question Explanation
(The complete setup for this game can be found here: https://forum.powerscore.com/lsat/viewtopic.php?t=3901)

The correct answer choice is (A)

The easiest guess for both questions #19 and #20 is G, because G is a variable that would prefer to perform earlier based on the second rule. However, during the game we would prefer you not to have to guess.

As discussed briefly at the end of question #19, G can perform at 6, and thus answer choice (A) is correct. Answer choice (B) can be eliminated because H in 6 violates the second rule. Answer choice (D) can be eliminated because Q in 6 violates the last rule. Answer choices (C) and (E) require further analysis.

Answer choices (C) and (E) involve two variables that are identical except for the fact that L is a folk band and T is a rock band. Otherwise, they operate in identical fashion. This basic similarity ultimately explains why neither can perform at 6.

If L or T performs at 6, then P must perform at 12. This leaves a set of four variables—G, H, Q, and the remainder of T/L—to fill all four of the performances at 8 and 10. If L performs at 6, the following scenario results:
D05_Game_#4_#20_diagram_1.png
Because of the second rule, G and H cannot perform at the same time, and thus G performs at 8 and H performs at 10. This leads to a violation of the third rule because only rock groups are available to perform with H, a folk group. Thus, L cannot perform at 6, and answer choice (C) is incorrect.

A similar result occurs when T performs at 6:
D05_Game_#4_#20_diagram_2.png

In this scenario, G works at 8 and H works at 10. Because T, a rock band, works at 6, from the last rule Q cannot work at 8, and Q must work at 10. However, if Q works with H a violation of the third rule occurs. Thus, T cannot work at 6, and answer choice (E) can be eliminated.

This inference works in the reverse as well: if L or T performs at 12, then P performs at 6, and the same variable set is available to fill the 8 and 10 performances. As this group cannot create a viable hypothetical, L and T also cannot perform at 12.
You do not have the required permissions to view the files attached to this post.

Get the most out of your LSAT Prep Plus subscription.

Analyze and track your performance with our Testing and Analytics Package.